PT61.S2.Q22 - psychiatrist: in treating first-year students

Tina ChoTina Cho Free Trial Member
edited August 2016 in Logical Reasoning 442 karma
Could anyone elaborate why B helps the argument?
I don't understand how "moderate" students are related in this case...thank you!!
https://7sage.com/lsat_explanations/lsat-61-section-2-question-22/

Comments

  • Accounts PlayableAccounts Playable Live Sage
    3107 karma
    Here's our argument:

    To help us treat freshmen, we have noticed that those reporting the highest levels of time spent doing fun things also score the same on depression/anxiety tests as those who report the lowest levels of time spent doing fun things. Thus, freshmen with a lot of time spent on fun things could spend less time doing those things without increasing their depression/anxiety.

    What I'm looking for: This is a pretty cookie cutter correlation to causation type argument, so to strengthen it, we need to deny all of the potential weakeners. Specifically, we need to deny an endogenous variable (something, Z, correlated to both depression levels and reported recreation time), deny that this correlation is a coincidence, and deny that the cause isn't reversed (or some structure more complicated than simply X causes Y). Another area of weakness in this argument has to do with "reporting;" we need to assume to that the freshmen were actually truthful in their statements.

    Since this is a strengthen EXCEPT question, 4 answer choices will strengthen while the correct answer will do nothing or weaken.

    Answer C is correct because we don't care about adults 40-60. The typical college freshmen is about 18 years old, so this isn't about the group we want.

    Answer B: This strengthens the argument because it lessens the likelihood that our X causes Y argument is simply a coincidence. This answer choice gives us more data to work with: if those in the middle show the same trend, then maybe we can expect those who reduce their their recreation time will also reduce their stress.
  • Tina ChoTina Cho Free Trial Member
    442 karma
    Hi thanks for your reply,
    So let me double check whether I understand your reasoning correctly... in the conclusion it says "...without increasing their anxiety or depression" so it doesn't say it will decrease anxiety or depression, but it could include this idea, and that's what B says.
    If the answer choice said it increases anxiety or depression, that would be against the conclusion, but the idea "decreasing" is part of the idea "without increasing" so B does not go against the conclusion.
    And moderate levels of spending of students can be those who were with the high levels of spending at first.
    (if students with high levels of spending reduce their spending, they will be moderate levels of spending of students...and if they reduce even, they will be the lowest levels of spending...it's continuum.). Thus, B supports the idea in the argument because 1) it shows reducing the spending will not increase but decrease their anxiety 2) add high→moderate data.
    Did I understand it correctly?
    Sorry if my writing is confusing...will appreciate further explanations. Thanks!
  • Tina ChoTina Cho Free Trial Member
    442 karma
  • Accounts PlayableAccounts Playable Live Sage
    3107 karma
    1.) Yes, that's what the conclusion means. "Without increasing" could mean staying the same or decreasing. If answer B had stated something about increasing anxiety, that would probably weaken the argument because it would be giving us data that shows the opposite effect in the conclusion.

    2.) Yes, that's part of what the "moderate" is implying. If we have data that suggests it's possible for people to have mid-level recreation and reduced depression, then that strengthens our relationship. Those students could have started out moderate or they could have been high recreation people and changed. Either way, it suggests the possibility that our relationship established by the data in the original fact pattern isn't out of left field.

  • Tina ChoTina Cho Free Trial Member
    442 karma
    Thanks bunch!! Now I see why B is not the answer here :)
Sign In or Register to comment.